Given the information in the passage, the author is LEAST likely to believe which one of the following?

jingjingxiao11111@gmail.com on April 11, 2020

Why is E wrong?

Hi Why is E wrong? Thanks

Replies
Create a free account to read and take part in forum discussions.

Already have an account? log in

shunhe on April 13, 2020

Hi jingjingxiao11111@gmail.com, sorry to keep bothering you about this, but some of the instructors are having troubles seeing the questions to which you refer. Do you happen to have the test/section/question number?

jingjingxiao11111@gmail.com on April 14, 2020

October 2002 SEC 3 Q12 Thanks